1answer.
Ask question
Login Signup
Ask question
All categories
  • English
  • Mathematics
  • Social Studies
  • Business
  • History
  • Health
  • Geography
  • Biology
  • Physics
  • Chemistry
  • Computers and Technology
  • Arts
  • World Languages
  • Spanish
  • French
  • German
  • Advanced Placement (AP)
  • SAT
  • Medicine
  • Law
  • Engineering
AnnyKZ [126]
3 years ago
12

Let p^1p^1 be the proportion of the 30 Americans sampled that agree climate change is an immediate threat to humanity.

Mathematics
1 answer:
pashok25 [27]3 years ago
8 0

Answer:

d. p^2p^2 is more likely to be greater than 0.3 than p^1p^1 is.

Step-by-step explanation:

We have two samples: one with n1=30 and a proportion p1, and the other with n2=100 and proportion p2.

We also know that the population is π=0.35.

As the sample size increase, the spread in the estimation of the true proportion is reduced. This is because the standard deviation of the sampling distribution decrease when the sample size increase.

Then, we would expect p2 to be closer to the true proportion than p1.

This is the only thing we can claim: p2 is more likely to be closer to the true population than p1.

Then, from the options available, we have:

a. p^1p^1 is more likely to be greater than 0.3 than p^2p^2 is.  

FALSE. p2 has an expected spread over the true population that is less than p1, so it is expected to be more close to 0.35, therefore bigger than 0.3, than p1.

b. p^1p^1 is more likely to be greater than 0.35 than p^2p^2 is.

FALSE. We can not claim that, because 0.35 is the population proportion and p1 and p2 can fall at either side of this value with equal probability.

c. p^1>p^2p^1>p^2

FALSE. p1 can be higher or lower than p2.

d. p^2p^2 is more likely to be greater than 0.3 than p^1p^1 is.

TRUE. As p2 is expected to be around the true proportion with less spread, is more likely to be greater than 0.3 than p1. It also will be more likely to be under 0.4 than p1.

You might be interested in
16x-8-14x=10 show work
kirill115 [55]

Answer:

x=9

Step-by-step explanation:

Original equation: 16x-8-14x=10

Combine like terms: 2x-8=10

Add 8 to both sides: 2x=18

Divide by 2 on both sides: x=9

Let me know if this helps!

5 0
3 years ago
Read 2 more answers
A pancake recipe require 1 2/3 cups of flour to make 20 pancakes and you have 9 cups of flour.
Leokris [45]
<h2>Solution (a) :-</h2>

Quantity of flour needed to make 20 pancakes =

= 1 \frac{2}{3}  \:

=  \frac{3  \times 1 + 2}{3}

=  \frac{5}{3}  \: cups \: of \: flour

Then , number of pancakes that can be made from 1 cup of floor :-

20 \div  \frac{5}{3}

20 \times  \frac{3}{5}

\frac{60}{5}

= 12 \: pancakes .

Therefore , 12 pancakes can be made from 1 cup of floor .

<h2>Solution (b) :-</h2>

Number of pancakes that can be made with 1 cup of flour = 12 pancakes

Number of pancakes that can be made with 9 cups of flour :-

= 9 × 12

= 108 pancakes

Therefore , I can make 108 pancakes with 9 cups of flour .

<h2>Solution (c) :-</h2>

Yes , I have enough flour to make 100 pancakes . I say this because :-

With one cup of flour I can make = 12 pancakes

With 9 cups of flour I can make =

= 9 × 12

= 108 pancakes

As I can make 108 pancakes with 9 cups of flour , I can conclude that I have enough flour to make 100 pancakes .

3 0
3 years ago
Which of the following graphs represents a function? If you don’t know then please don’t write “i don’t know” or “I can’t help w
Vadim26 [7]

Answer:

B. x

Step-by-step explanation:

The answer would be graph x. A function must be that one x value should only have one y value. graph w doesn't work since when you plug in 0, you get two y's. same thing with z. Y doesn't work since at x value of 3, it curves back so x value 2 has 2 y's. X is the only one that each x value has only one y.

6 0
3 years ago
Read 2 more answers
Please help! This is for Algebra 1
kow [346]
The first answer
-40y3/x5
8 0
3 years ago
Penelope went to the store to buy x pounds of broccoli for $1.10 per pound and y cans of soup for $2.50 each. In total, she spen
kati45 [8]
2.5y + 1.1x ∠ 10

2.5y = - 1.1x +10

y = (- 1.1/2.5)x + 4

Draw this function. It's descending (m negative). All values on the left of the lines satisfy this inequality


6 0
4 years ago
Other questions:
  • How do you solve 1 1/9
    10·1 answer
  • Cosx=5/8
    8·1 answer
  • How to solve h when equation is |3h+1|=7h
    11·1 answer
  • I need to solve this equation
    5·2 answers
  • Number 5 says: a punch recipe that serves 24 people calls for 4 liters of lemon-lime soda,2 pints of sherbet and 6 cups of ice.
    12·1 answer
  • 3x^2 - 4x + 2 = 0 <br> how many solutions does the equation above have?
    6·1 answer
  • Can anyone write this in a notebook for more understanding? Thank you
    7·1 answer
  • Find the loan-to-value ratio for a home appraised at $583,620.00 that the buyer will purchase for $585,000.00. The buyer plans t
    5·1 answer
  • Evaluate the expression 4^3+(96/8)*5)-48 =
    13·1 answer
  • A person places $308 in an investment account earning an annual rate of 2.1%,
    15·1 answer
Add answer
Login
Not registered? Fast signup
Signup
Login Signup
Ask question!